Looking for proof of Superpositional Energy Conservation

In summary: ), which can be explained by the mutual interference of the waves from each point source, as well as by the fact that the waves from each point source add together only when they are in phase.
  • #1
greswd
764
20
Let's have waves with their carried power proportional to the square of their amplitude(s).

The waves obey the principle of superposition.

Before superposition, we can calculate the power output based on the amplitudes.
After superposition, there will be new values for the amplitudes, but the power output is expected to remain the same, by the Law of CoE.

I'm looking for a mathematical proof which shows that the superposition of such waves is consistent the Law of CoE.
 
Science news on Phys.org
  • #2
The question you are asking needs a dynamical model for the waves. So you must tell us which waves you are talking about. Usually you start from a Lagrangian. Then Noether's theorem tells you how the energy-momentum tensor looks and provides the energy-balance equation.

E.g. take elecromagnetic waves in a vacuum. In Heaviside-Lorentz units the energy density is
$$\epsilon=\frac{1}{2}(\vec{E}^2+\vec{B}^2).$$
Together with the Poynting vector (energy flow density),
$$\vec{S}=c \vec{E} \times \vec{B}$$
you get the energy balance equation
$$\partial_t \epsilon + \vec{\nabla} \cdot \vec{S}=-\vec{j} \cdot \vec{E}.$$
On the left-hand side you have the temporal change of the energy density and the flow of energy out of the surface of the corresponding volume element, and on the right-hand side the energy lost by the field due to mechanical work on the charges. The total energy (field energy + mechanical energy of the charges) is conserved.
 
  • #3
vanhees71 said:
The question you are asking needs a dynamical model for the waves. So you must tell us which waves you are talking about. Usually you start from a Lagrangian. Then Noether's theorem tells you how the energy-momentum tensor looks and provides the energy-balance equation.

E.g. take elecromagnetic waves in a vacuum. In Heaviside-Lorentz units the energy density is
$$\epsilon=\frac{1}{2}(\vec{E}^2+\vec{B}^2).$$
Together with the Poynting vector (energy flow density),
$$\vec{S}=c \vec{E} \times \vec{B}$$
you get the energy balance equation
$$\partial_t \epsilon + \vec{\nabla} \cdot \vec{S}=-\vec{j} \cdot \vec{E}.$$
On the left-hand side you have the temporal change of the energy density and the flow of energy out of the surface of the corresponding volume element, and on the right-hand side the energy lost by the field due to mechanical work on the charges. The total energy (field energy + mechanical energy of the charges) is conserved.
Let's ignore charges for this problem.
Consider
$$(\vec{E_1} \times \vec{B_1})+(\vec{E_2} \times \vec{B_2})$$
versus
$$(\vec{E_1}+\vec{E_2}) \times (\vec{B_1}+\vec{B_2})$$That's my naive interpretation of the Poynting vector before and after superposition, which is most probably wrong.

So I'm trying to figure out how energy is still conserved after superposition.
 
  • #4
greswd said:
I'm looking for a mathematical proof which shows that the superposition of such waves is consistent the Law of CoE.
Maxwell's equations are linear, so they obey superposition. And from Maxwell's equations we obtain Poynting's theorem, which shows energy conservation.

What more mathematical proof can you have?
 
  • #5
greswd said:
Consider
$$(\vec{E_1} \times \vec{B_1})+(\vec{E_2} \times \vec{B_2})$$
This is not a relevant quantity. Poynting's theorem holds for Maxwell's equations, but this quantity is not related to it.
 
  • #6
Dale said:
This is not a relevant quantity. Poynting's theorem holds for Maxwell's equations, but this quantity is not related to it.
what if we only have information about two Poynting vector fields, with no information regarding the E and B vector fields?

how should we superpose those two Poynting vector fields?
 
  • #7
In the Poynting vector and the energy density you have the electromagnetic field components. Why should you split them somehow in two parts? It's always the full electromagnetic field from all sources entering. Maxwell's equations are linear and thus the superposition principle holds, i.e., the electromagnetic field from different sources add simply up, but that cannot hold true for the energy density and Poynting vector since these are products of field components.
 
  • Like
Likes Charles Link
  • #8
greswd said:
what if we only have information about two Poynting vector fields, with no information regarding the E and B vector fields?

how should we superpose those two Poynting vector fields?
You cannot. If you don't have enough information then you don't have enough information
 
  • Like
Likes vanhees71
  • #9
I think the OP is asking a very good question. With multiple occupancy of the Boson cavity modes (at the same frequency, e.g. as in a laser), if you assume the photon phases are all the same, it leads to a complete contradiction. One explanation that provides for superposition along with energy conservation is that the phases of individual photons go in randomly. (realizing of course that it is somewhat unsound to consider the phase of a single photon.) The phasor diagram becomes a 2-D random walk with the resultant E proportional to ## \sqrt{N} ## (for large ## N ##) where ## N ## is the number of photons. Since energy is proportional to E^2, it is then proportional to photon number. ## \\ ## The above is for plane wave sources that lay right on top of each other. In the case of mutually coherent point sources at separate spatial locations, the intensity patterns do not superimpose, (and they are not required to=the intensity is second order in the linear parameters (E,B) of the Maxwell equations=thereby they are not required to obey superposition and linear properties), but instead interference patterns necessarily result that conserve total energy. ## \\ ## And finally, in the case of mutually incoherent sources, the intensity patterns do superimpose in a linear fashion.
 
Last edited:
  • #10
vanhees71 said:
In the Poynting vector and the energy density you have the electromagnetic field components. Why should you split them somehow in two parts? It's always the full electromagnetic field from all sources entering. Maxwell's equations are linear and thus the superposition principle holds, i.e., the electromagnetic field from different sources add simply up, but that cannot hold true for the energy density and Poynting vector since these are products of field components.
I agree. I made a similar statement in post #9 that the intensity functions of the sources are not required to obey linear principles. Perhaps this comes as somewhat of a surprise to some, especially when they are working with equations (Maxwell's) that are completely linear. The system is said to be "completely linear" even though some features (e.g. intensity patterns) can appear that do not behave linearly. Perhaps it is a "misnomer" to call it a "completely linear" system because the energy equations of the system are not linear (but are in fact quadratic in the E and B fields).
 
Last edited:
  • #11
Charles Link said:
think the OP is asking a very good question. With multiple occupancy of the Boson cavity modes
The OP is asking a purely classical question. I see no value in bringing quantum stuff into the discussion.
 
  • #12
Charles Link said:
The system is said to be "completely linear" even though some features (e.g. intensity patterns) can appear that do not behave linearly. Perhaps it is a "misnomer" to call it a "completely linear" system because the energy equations of the system are not linear (but are in fact quadratic in the E and B fields).
I disagree completely. "Linear" has a well defined meaning, and the fields meet that definition, therefore they are linear.

You can always take any linear equation, do some mathematical operation on it and get something nonlinear. So if that were the requirement, then we could never call anything "linear". We might as well not even use the word.
 
  • #13
Dale said:
The OP is asking a purely classical question. I see no value in bringing quantum stuff into the discussion.
The problem can be considered in a very classical context. Considering the energy requirements, the question arises, is it possible to overlay two identical macroscopic electromagnetic plane waves? It would appear the OP is also asking this question, and that question has a slightly more mathematically complex but completely classical solution. The apparatus (using optical interferometers) exists to perform such a superposition of two plane waves with complete conservation of energy, and the explanation of the physics involves the fresnel coefficients. The solution I gave when the question involves many, but individual photons, I think also deserves an answer, which I gave.
Dale said:
I disagree completely. "Linear" has a well defined meaning, and the fields meet that definition, therefore they are linear.

You can always take any linear equation, do some mathematical operation on it and get something nonlinear. So if that were the requirement, then we could never call anything "linear". We might as well not even use the word.
I was just pointing out a subtlety. The word "linear" will continue to be used for such systems, and I, of course, also use the word.
 
Last edited:
  • #14
Just an additional item of interest. (The discussion hasn't had any additional responses, but if anyone is still following it they may find it of interest.) When my classmates and I were first presented with the Fraunhofer diffraction derivation of the two slit and multi-slit problem, it was presented as a sequence of mathematical steps to arrive at the intensity pattern. It wasn't taught as being a linear process, and the energy part of it is not a linear process. We learned the derivations in detail, but it wasn't until later that this whole process was shown to come from Maxwell's equations, with the E-fields and B-fields being the linear parameters of these linear (Maxwell's) equations. The interference patterns, which exhibit some properties that are not of a linear nature, is a result of the energy expressions being second order in the linear parameters rather than first order.
 
  • #15
Charles, I'm rather fearful that you might derail this thread. Your verbosity makes it harder for others, sorry to say.
 
  • Like
Likes Vanadium 50
  • #16
greswd said:
Charles, I'm rather fearful that you might derail this thread. Your verbosity makes it harder for others, sorry to say.
To each their own. I can't please everybody. I will refrain from further comment on your thread.
 
  • #17
Charles Link said:
To each their own. I can't please everybody. I will refrain from further comment on your thread.
Thank you for your understanding.
 
  • #18
Dale said:
What more mathematical proof can you have?

chill bro, I need some time to wrap my head around it.I was thinking of the double-slit experiment.

interference%202.jpg

interference1.gif


In this diagram you can see the nodal and anti-nodal lines, the bright and dark fringes of an interference pattern:

nodal_lines-gif.80105.gif


The nodal lines are hyperbolas.

If we consider distances far, far away from the sources, the hyperbolas can be approximated as radiating from one specific point.I created a Desmos worksheet to demonstrate a plot of the Intensity against angle:
https://www.desmos.com/calculator/jv5sk0mucx

It follows this formula:
l9ZBCYU.png


Now consider a circular screen which is extremely large so we can do the radial approximation. We also don't have to worry about the inverse-square law because the circular screen makes all the distances about the same. I assume the energy output is approximated by integrating the intensity across the arc distance of the circular screen.

If you try integrating the formula above from π/2 to 0, you get this:

https://www.wolframalpha.com/input/?i=int+(cos((a*tanx)/sqrt(1+(tanx)^2)))^2+from+0+to+pi/2

Based on my naive assumptions about energy conservation, I assume that energy is transferred from dark fringes to bright fringes.

I thus expect the integral to be simply π/4. Because looking at my Desmos plot, I expect the area under the green curve to be equal to the area under the blue horizontal line if energy is conserved.However, from Wolfram Alpha, the integral is not simply π/4, but has a Bessel function component. This is what I'm confused about.

d represents the slit spacing and W_L represents the wavelength. Apologies for any grammatical errors, I'm quite tired as I'm writing this. Feel free to ask me any questions about the Desmos worksheet.
 
  • #19
greswd said:
Based on my naive assumptions about energy conservation, I assume that energy is transferred from dark fringes to bright fringes.

I thus expect the integral to be simply π/4.
OK, this seems less like a problem with EM superposition and more like a problem with EM energy conservation.

The conservation of energy in EM is given by Poynting's theorem. What does Poynting's theorem actually say? How would you apply it to a given field to check if energy is conserved?
 
Last edited:
  • #20
Dale said:
The conservation of energy in EM is given by Poynting's theorem. What does Poynting's theorem actually say? How would you apply it to a given field to check if energy is conserved?
Hmm, Poynting's theorem talks about the work done on charges, which aren't present in the double-slit setup.

I thought of Poynting flux and dot products, but at large distances the Poynting or P-vectors are all close to being perpendicular to the screen. If we assume that all the E vectors are pointing in the same direction, at large distances the B vectors of waves from both slits are close to parallel at the point where they meet on the screen, the vector sum would be the sum of their magnitudes.

I'm still wondering why there's a Bessel component.
 
  • #21
there is a slight mistake in the formula I posted. let me correct it.
 
  • #22
here's the new Desmos link: https://www.desmos.com/calculator/8gcwydxb4t

In the top left corner, we can see that for a slit-spacing 9.5 times that of the wavelength, about 7.3% of the energy is "lost". For other ratios, there may be a "gain" instead.
 
  • #23
greswd said:
Hmm, Poynting's theorem talks about the work done on charges, which aren't present in the double-s
OK, that is one of the terms in Poyntings theorem. We set that term to 0 in vacuum.

What about the other terms. What do they say and how would you check to see if they conserve energy?
 
  • #24
Dale said:
OK, that is one of the terms in Poyntings theorem. We set that term to 0 in vacuum.

What about the other terms. What do they say and how would you check to see if they conserve energy?
ok, I can see how it relates to the rate of change of energy density.

but idk how I should modify the model in #22.
 
  • #25
greswd said:
ok, I can see how it relates to the rate of change of energy density.

but idk how I should modify the model in #22.
The model in 22 does not have enough information. You need to know the fields, not just the intensity. Then you can evaluate Poynting's theorem and confirm energy conservation.
 
  • #26
Sorry for my late replies, I'm pretty busy.

Dale said:
The model in 22 does not have enough information. You need to know the fields, not just the intensity. Then you can evaluate Poynting's theorem and confirm energy conservation.
As I mentioned earlier:

greswd said:
I thought of Poynting flux and dot products, but at large distances the Poynting or P-vectors are all close to being perpendicular to the screen. If we assume that all the E vectors are pointing in the same direction, at large distances the B vectors of waves from both slits are close to parallel at the point where they meet on the screen, the vector sum would be the sum of their magnitudes.
Also, I realized that when the slit spacing 'd' becomes arbitrarily small, both sources converge towards behaving like a single source with double the amplitude and with double the power output, which sounds very paradoxical.
 
  • #27
greswd said:
I realized that when the slit spacing 'd' becomes arbitrarily small, both sources converge towards behaving like a single source with double the amplitude and with double the power output, which sounds very paradoxical.
But the slit spacing cannot be arbitrarily small. In fact, it cannot be less than the slit width.
 
  • #28
Dale said:
But the slit spacing cannot be arbitrarily small. In fact, it cannot be less than the slit width.
Is it okay to assume point sources? meaning arbitrarily small slit widths.
 
  • #29
greswd said:
Is it okay to assume point sources? meaning arbitrarily small slit widths.
Sure, but then the energy is 0 so double 0 is still 0.
 
  • #30
Dale said:
Sure, but then the energy is 0 so double 0 is still 0.
What about infinitesimally small slit widths and slit spacings? The energy is near zero but the ratio of superposed vs un-superposed energy remains the same.

If we really need to abandon infinitesimally small slit widths, we can just increase the wavelength used, as the far-field result largely depends on the slit-spacing to wavelength ratio.

For a slit-spacing and wavelength ratio of 1, the superposed area is 22% larger than the un-superposed one. That's a pretty large discrepancy when we're trying to make the case that there should be no discrepancy at all.
 
Last edited:
  • #31
greswd said:
If we really need to abandon infinitesimally small slit widths
It is an approximation. It is never "true", but it can be "valid" (meaning "close enough"). Its validity depends entirely on what you want to use it for, and when you get a clearly wrong answer it is always good to go back and check your approximations. For the purpose of post 26, the approximation gives an answer that is no longer "close enough", so it is not a valid approximation.

greswd said:
For a slit-spacing and wavelength ratio of 1, the superposed area is 22% larger than the un-superposed one. That's a pretty large discrepancy when we're trying to make the case that there should be no discrepancy at all.
What are you referring to here? What is this superposed area and what is the case are you trying to make?
 
Last edited:
  • #32
greswd said:
What are you referring to here? What is this superposed area and what is the case are you trying to make?

The area under the intensity function that I first mentioned in #18.

You can play with the Desmos widget too:
https://www.desmos.com/calculator/8gcwydxb4t
 
  • #33
greswd said:
The area under the intensity function that I first mentioned in #18
Again neither the intensity nor the Poynting vector obey superposition. As far as I understand what you are doing, the claim that there is no discrepancy is false, and it is not a case that I am trying to make.

The fields obey Maxwell's equations, which is linear and therefore the fields obey superposition. The fields also conserve energy, per Poynting's theorem. The intensity is the magnitude of the Poynting vector, so it is only a part of the energy conservation equation. Furthermore, the Poynting vector itself does not obey superposition.

Superposition and energy conservation are separate concepts.
 
  • #34
Dale said:
Again neither the intensity nor the Poynting vector obey superposition. The claim that there is no discrepancy is false, and it is not a case that I am trying to make.

The fields obey Maxwell's equations, which is linear and therefore the fields obey superposition. The fields also conserve energy, per Poynting's theorem. The intensity is the magnitude of the Poynting vector, so it is only a part of the energy conservation equation. Furthermore, the Poynting vector itself does not obey superposition.

Superposition and energy conservation are separate concepts.
If the intensity is only a part of the energy conservation equation, what's the other part?
 
  • #35

1. What is Superpositional Energy Conservation?

Superpositional Energy Conservation is a scientific principle that states the total energy in a closed system remains constant over time, regardless of any changes that may occur within the system.

2. How is Superpositional Energy Conservation related to quantum mechanics?

Superpositional Energy Conservation is a concept that arises from the principles of quantum mechanics, which describe the behavior of particles at the subatomic level. It is a fundamental law that governs the behavior of energy at the quantum level.

3. Is there any evidence for Superpositional Energy Conservation?

Yes, there is strong evidence for Superpositional Energy Conservation. This principle has been tested and confirmed through numerous experiments and observations in the field of quantum mechanics.

4. How does Superpositional Energy Conservation impact our daily lives?

Superpositional Energy Conservation has a significant impact on our daily lives, as it governs the behavior of energy at the subatomic level. This principle is important in understanding and developing technologies such as transistors, lasers, and computer memory.

5. Are there any exceptions to Superpositional Energy Conservation?

There are currently no known exceptions to Superpositional Energy Conservation. However, it is important to note that this principle only applies to closed systems, meaning that energy can be transferred or transformed within the system, but the total energy remains constant.

Similar threads

  • Quantum Physics
Replies
1
Views
956
  • Astronomy and Astrophysics
Replies
13
Views
1K
Replies
57
Views
3K
  • Quantum Interpretations and Foundations
Replies
5
Views
1K
  • Electrical Engineering
Replies
7
Views
2K
  • Special and General Relativity
Replies
21
Views
2K
  • Mechanics
2
Replies
53
Views
2K
  • Introductory Physics Homework Help
Replies
22
Views
2K
Back
Top